CARDIO Flashcards

TO PASS

1
Q

1 Why do we NOT see atrial repolarization in a normal ECG tracing?
A Because it is buried in the concurrent QRS complex
B Because it is isoelectric and hence cannot be seen
C Because it is of too low voltage to be detected by the ECG
D Because it occurs together with ventricular repolarization

A

A Because it is buried in the concurrent QRS complex

How well did you know this?
1
Not at all
2
3
4
5
Perfectly
2
Q
When recording lead I on an ECG, the right arm is the negative electrode. Where is the positive electrode placed?"
A Left arm
B Left leg
C Right leg
D Left foot
A

A Left arm

How well did you know this?
1
Not at all
2
3
4
5
Perfectly
3
Q
3 Quantity of blood that goes to the right atrium, which comes from all the peripheral veins, is called 
A Venous return
B Cardiac output
C Stroke volume
D End-diastolic volume
A

A Venous return

How well did you know this?
1
Not at all
2
3
4
5
Perfectly
4
Q

4 Which electrophysiologic event in the cardiac cycle is CORRECTLY matched with its counterpart in ECG?
A Ventricular repolarization – QRS complex
B Atrial repolarization – P wave
C Ventricular depolarization and repolarization – QT interval
D PR interval – conduction from atria to bundle of His

A

C Ventricular depolarization and repolarization – QT interval

How well did you know this?
1
Not at all
2
3
4
5
Perfectly
5
Q
5 Which heart sound is caused by the closure of the semilunar valves which marks the end of systole?
A S4
B S1
C S3
D S2
A

D S2

How well did you know this?
1
Not at all
2
3
4
5
Perfectly
6
Q
6 Increase in which ion's concentration locally causes vasoconstriction?
A Potassium
B Calcium
C Magnesium
D Acetate
A

B Calcium

How well did you know this?
1
Not at all
2
3
4
5
Perfectly
7
Q

7 Why do we NOT use lead aVR for the assessment of ECG changes pertaining to anatomical walls of the heart?
A Because it is NOT part of Einthoven’s triangle
B Because it is perpendicular to the cardiac apex
C Because it almost always equiphasic
D Because it does NOT face the epicardial surface of the heart

A

D Because it does NOT face the epicardial surface of the heart

How well did you know this?
1
Not at all
2
3
4
5
Perfectly
8
Q
8 Ventricular volume stays constant while ventricular pressure rises during which phase of the cardiac cycle?
A Isovolumic contraction
B Pre-systole
C Isovolumic relaxation
D Diastasis
A

A Isovolumic contraction

How well did you know this?
1
Not at all
2
3
4
5
Perfectly
9
Q
9 "A 45/M suffers from an acute ST elevation myocardial infarction involving the lateral wall of the left ventricle. Using your knowledge of the anatomic orientation of ECG limb leads, in which of the following leads will you MOST likely detect abnormalities?"
A Leads V1 and V2
B "Leads II, III and aVF"
C "Leads I, aVL, V5 and V6"
D Leads V3 and V4
A

C “Leads I, aVL, V5 and V6”

How well did you know this?
1
Not at all
2
3
4
5
Perfectly
10
Q

10 Which of the following would MOST likely occur if the papillary muscles are paralyzed?
A There will be no ventricular filling during diastole
B The semilunar valves would stay open throughout the cardiac cycle
C The leaflet attached to the chorda bulges towards the atrium allowing backflow of blood
D The semilunar valves would become stenotic

A

C The leaflet attached to the chorda bulges towards the atrium allowing backflow of blood

How well did you know this?
1
Not at all
2
3
4
5
Perfectly
11
Q
11 Increased heart rate increases the force of cardiac contraction in a stepwise fashion. What mechanism is this?
A Post extrasystolic potentiation
B Bowditch phenomenon
C Frank-Starling's mechanism
D Bainbridge reflex
A

B Bowditch phenomenon

How well did you know this?
1
Not at all
2
3
4
5
Perfectly
12
Q
12  What is the most important of the local vasodilators that regulate local blood flow?
A Adenosine
B Nitric oxide
C Carbon dioxide
D Hydrogen ions
A

A Adenosine

How well did you know this?
1
Not at all
2
3
4
5
Perfectly
13
Q
13 "Which of the following represents time from end of ventricular depolarization to the start of ventricular repolarization? 
A PR interval
B QT interval
C QRS complex
D ST segment
A

D ST segment

How well did you know this?
1
Not at all
2
3
4
5
Perfectly
14
Q
14 Most important driver of changes in local blood flow is the concentration of \_\_\_\_\_\_\_\_\_\_\_\_\_\_\_\_\_\_\_\_\_\_\_\_.
A Waste metabolites
B Carbon dioxide
C B vitamins
D Oxygen
A

D Oxygen

How well did you know this?
1
Not at all
2
3
4
5
Perfectly
15
Q
15 "When the volume of blood entering the aorta during rapid ejection exceeds peripheral run-off, which of the following event is expected?"
A Aortic pressure remains unchanged 
B Aortic pressure goes down
C Aortic pressure increases 
D venous return increases
A

C Aortic pressure increases

How well did you know this?
1
Not at all
2
3
4
5
Perfectly
16
Q
16 Dina has been diagnosed with rheumatic heart disease and has been told that her mitral valve does not open completely. This means her valve is 
A Regurgitant
B Stenotic
C Ischemic 
D Degenerated
A

B Stenotic

How well did you know this?
1
Not at all
2
3
4
5
Perfectly
17
Q

17 Which of the following statements about arterial blood pressure is FALSE?
A The systolic pressure is the pressure maintained by the recoil of the stretched arterial walls during ventricular relaxation
B It is higher in the long systemic arteries than in the short pulmonary circuit
C The diastolic pressure is the pressure generated during relaxation of the ventricles
D Its maintenance is dependent on total peripheral resistance and cardiac output

A

A The systolic pressure is the pressure maintained by the recoil of the stretched arterial walls during ventricular relaxation

How well did you know this?
1
Not at all
2
3
4
5
Perfectly
18
Q
18 Vasoconstriction of periperal arterioles occur as a response to hemorrhage EXCEPT in which vascular bed?
A Hepatic 
B Renal
C Cerebral
D Gastrointestinal
A

C Cerebral

How well did you know this?
1
Not at all
2
3
4
5
Perfectly
19
Q
19 "Total amount of blood that can be stored in a given portion of the circulation per ml of mercury rise in pressure is \_\_\_\_\_\_\_\_\_\_\_\_\_\_\_\_\_\_\_\_\_\_ "
A Resistance
B Distensibility
C Compliance
D Cardiac index
A

C Compliance

How well did you know this?
1
Not at all
2
3
4
5
Perfectly
20
Q
20 One of the chronic changes in regulating blood flow that happen in congestive heart failure is \_\_\_\_\_\_\_\_\_\_\_\_\_\_\_\_\_\_\_?
A Renal retention of fluid
B Sympathetic stimulation
C Baroreceptor reflex
D Chemoreceptor reflex
A

A Renal retention of fluid

How well did you know this?
1
Not at all
2
3
4
5
Perfectly
21
Q

21 Which of the following pairs correctly describes microcirculation at the capillary level?
A Kidneys: Wide gaps for plasma proteins
B Brain: tight junctions
C Intercellular clefts: lipid-soluble substances
D GI tract: capillary fenestrations

A

B Brain: tight junctions

How well did you know this?
1
Not at all
2
3
4
5
Perfectly
22
Q

22 “During a routine physical examination of a 14 year old male high school student, a third heart sound is heard . S3 is associated with which of the following?”
A Atrial systole as the ventricle fills with blood
B The rapid in rush of blood during rapid ventricular filling
C The closure of the AV valves
D Delayed closure of the semilunar valves

A

B The rapid in rush of blood during rapid ventricular filling

How well did you know this?
1
Not at all
2
3
4
5
Perfectly
23
Q
23 Which of the following types of congenital heart disease presents with cyanosis at birth?
A Chamber/valve atresia
B Left to right shunt
C Stenotic heart/vascular channel
D Right to left shunt
A

D Right to left shunt

How well did you know this?
1
Not at all
2
3
4
5
Perfectly
24
Q

24 “How does a high concentration of potassium in the ECF, such as in renal failure, affect the function of the heart?”
A An increase in cardiac cell excitability
B An increase in cardiac rate
C An increase in myocardial contractility
D slow flaccid heart

A

D slow flaccid heart

How well did you know this?
1
Not at all
2
3
4
5
Perfectly
25
Q

25 In which of the following situations will stroke volume increase?
A When one exhales against a closed glottis
B When the heart is highly tachycardic
C When ventricular compliance is increased
D When the aortic valve becomes stenosed

A

C When ventricular compliance is increased

How well did you know this?
1
Not at all
2
3
4
5
Perfectly
26
Q

26 Rapid arrhythmias cause reduction in cardiac output because of ______________________
A Reduced diastolic filling
B Reduced end-systolic volume
C Uncoordinated atrial and ventricular contractions
D Increased myocardial wall stretch

A

A Reduced diastolic filling

How well did you know this?
1
Not at all
2
3
4
5
Perfectly
27
Q
27 An increase in blood hematocrit will correspondingly increase which of the following?
A Total peripheral resistance
B Blood flow velocity
C Venous return
D Cardiac output
A

A Total peripheral resistance

How well did you know this?
1
Not at all
2
3
4
5
Perfectly
28
Q
28 "Coronary imaging done in a 56 year-old obese female who recently had a myocardial infarction showed a block in the left anterior descending artery and some of its branches. During your rounds with the attending physician, he asked you, a medical student, to locate the infarction on her EKG. Where will you most likely see the changes?"
A  "I, aVL, V5, V6"
 B "II, III, aVF"
C "V2, V3"
D "I, aVF"
A

C “V2, V3”

How well did you know this?
1
Not at all
2
3
4
5
Perfectly
29
Q

29 What is the usual pressure in the right atrium in mmHg?

A 0
B 14
C 6
D -10

A

D -10

How well did you know this?
1
Not at all
2
3
4
5
Perfectly
30
Q

30 How does a high concentration of calcium in the ECF affect the function of the heart?
A It has no effect on the activity of the SA node
B It tends to make the heart muscle spastic
C The action potential decreases and the heart muscle becomes flaccid
D It hyperpolarizes the cardiac tissue

A

B It tends to make the heart muscle spastic

How well did you know this?
1
Not at all
2
3
4
5
Perfectly
31
Q
31 Which of the following is directly proportional to the rate of diffusion across capillaries?
A  Lipid solubility
B Solute molecular weight
C Concentration gradient
D Water solubility
A

C Concentration gradient

How well did you know this?
1
Not at all
2
3
4
5
Perfectly
32
Q

32 Which of the following BEST explains why the semilunar valves CLOSE during a normal cardiac cycle?
A Ventricular pressure becomes less than aortic pulmonary pressure
B Ventricular volume becomes greater than atrial volume
C Ventricular pressure becomes greater than aortic and pulmonary pressure
D Ventricular volume becomes less than atrial volume

A

A Ventricular pressure becomes less than aortic pulmonary pressure

How well did you know this?
1
Not at all
2
3
4
5
Perfectly
33
Q
33 A 67 year-old male with chronic coronary insufficiency suffers from an acute ST elevation myocardial infarction. ST elevations are appreciated in leads V5 and V6. Which among the following coronary vessels is MOST likely affected?
A Left posterior descending artery
B Left anterior descending artery
C Right coronary artery
D Left circumflex artery
A

D Left circumflex artery

How well did you know this?
1
Not at all
2
3
4
5
Perfectly
34
Q
34 In which site would you find the highest concentration of proteins in lymph?
A Skeletal muscles
B GI tract
C Kidneys
D  Liver
A

D Liver

How well did you know this?
1
Not at all
2
3
4
5
Perfectly
35
Q

35 What happens to blood flow to the eyes when oxygen levels decrease?
A Vasoconstriction due to endothelin
B Vasodilation due to inability to contract vascular smooth muscle
C Vasodilation due to closure of the metarteriole
D Vasoconstriction due to potassium entry to smooth muscle

A

B Vasodilation due to inability to contract vascular smooth muscle

How well did you know this?
1
Not at all
2
3
4
5
Perfectly
36
Q
36 Mrs D has ongoing vaginal bleeding. Which of the following makes sure that her blood does NOT pool in the skeletal muscles and goes to other vital organs instead?
A Sympathetic vasoconstrictor fiber
B Vagal acetylcholine receptors
C Atrial stretch fibers
D Skeletal baroreceptors
A

A Sympathetic vasoconstrictor fiber

How well did you know this?
1
Not at all
2
3
4
5
Perfectly
37
Q

37 What is the major contributor to after load in the heart?
A The state of the semilunar valves
B The resting length from which the muscle contracts
C Aortic pressure
D Vena caval pressure

A

C Aortic pressure

38
Q

38 Which of following will facilitate ventricular filling?
A An increase in central venous pressure and a more negative intrathoracic pressure
B A decrease in central venous pressure and an increase in intrathoracic pressure
C A decrease in intrathoracic pressure and a decrease in central venous pressure
D An increase in central venous pressure and an increase intrathoracic pressure

A

A An increase in central venous pressure and a more negative intrathoracic pressure

39
Q
39 Which of the following causes vasodilation of local and upstream arteries due to increased shear stress in the vessel wall?
A Bradykinin
B Histamine
C  Endothelium-derived relaxing factor
D Angiogenin
A

C Endothelium-derived relaxing factor

40
Q

40 The ““C”” wave of the atrial pressure pulse is produced by which of the following?”
A Over bulging of the AV valves during ventricular systole
B Recoil of the aorta during ventricular diastole
C Over bulging of the AV valves during ventricular diastole
D Atrial filling during reduced ejection

A

A Over bulging of the AV valves during ventricular systole

41
Q
41 The CNS ischemic response does NOT become activated until the arterial BP goes below \_\_\_\_\_\_\_\_\_\_\_\_\_\_\_\_
A 70 mmHg
B 80 mmHg
C 50 mmHg
D 60 mmHg
A

D 60 mmHg

42
Q

42 Which of the following is / are TRUE of the cardiac valves?
A All choices are correct
B They are found in the entrance of all big vessels
C They moves actively as they contract
D They close or open passively as a result of pressure gradient on either side

A

D They close or open passively as a result of pressure gradient on either side

43
Q
43  "Which EKG waveform represents the time required for SA node impulse to travel through the conduction system? 
A P interval
B PR  wave
C QT interval
D T wave
A

B PR wave

44
Q
44 "What is the ionic basis for recording electrical activity of the heart using electrodes placed on skin surface, making it a good conductor of electricity?"
A Na+ and K+ in ECF
B Na+ and Cl- in ECF
C Na+ and HCO3- in ICF
D Na+ and Cl- in ICF
A

B Na+ and Cl- in ECF

45
Q

45 Name one reason for increased risk of ventricular fibrillation after a myocardial infarction
A Increased parasympathetic tone
B Rapid depletion of calcium
C Injury current in infarcted tissue
D Decreased pathway length for conduction

A

C Injury current in infarcted tissue

46
Q

46 Dynamic exercises like swimming produces a little change in mean arterial pressure because the increase in cardiac output is balanced by a decrease in systemic vasculature resistance. Which of the following statements is/are correct?
A Diastolic pressure is lower because of the fall in systemic vasculature resistance allowing greater peripheral run-off
B Systolic pressure will increase because of elevated stroke volume
C There will be an increase in pulse pressure
D All choices are correct

A

D All choices are correct

47
Q
47 In what region of the AV node is AV block more likely to occur?
A H region
B AN region
C N region
D NH region
A

C N region

48
Q
48 What is the major determinant of cardiac output?
A Heart  Rate
B Stroke Volume
C Venous Return 
D Blood Pressure
A

C Venous Return

49
Q
49 "Which of the following can result to swelling of the feet, hands and abdomen?"
A Decline in capillary permeability
B Overproduction of plasma proteins
C Decreased peripheral venous pressure
D Lymphatic blockage
A

D Lymphatic blockage

50
Q
50  Vasomotion is regulated by which of the following part of the microcirculation?
A Arteriole
B Postcapillary sphincter
C Venule
D Metarteriole
A

D Metarteriole

51
Q
51 Which of the following receives the stimulus for the chemoreceptor reflex?
A Carotid sinus
B Carotid bodies
C Aortic arch receptors
D Atrial receptors
A

B Carotid bodies

52
Q

52 The positive inotropic effect of cardiac glycosides is explained by which of the following?
A Its inhibitory effect on the Na+/K+ ATPase
B Its stimulatory effect on the Ca++-Na+ exchange
C Its stimulatory effect on the Na+/K+ ATPase
D Its stimulatory effect on the inward sodium current

A

A Its inhibitory effect on the Na+/K+ ATPase

53
Q
53 "In EKG, which lead/s represent/s a view of the lateral wall from the left arm?"
A "I, aVL"
B "II, III, aVF"
C "V1, V2, V3"
D "V5, V6"
A

A “I, aVL”

54
Q
54 At which phase of the nodal action potential do calcium channels open?
A Phase 4
B Phase 0
C beginning of Phase 3
D Phase 3
A

B Phase 0

55
Q

55 Which among the following BEST describes the physiologic function of the slow conduction velocity through the AV node?
A It allows atrial contraction to be completed before ventricular contraction
B It favors the peripheral run-off of blood during ventricular diastole
C It allows sufficient time for venous return to the heart
D It allows sufficient time for ventricular repolarization

A

A It allows atrial contraction to be completed before ventricular contraction

56
Q
56 What is the most important and potent stimulus for cerebral blood flow regulation?
A Increase in HCO3-
B Decrease in O2
C Increase in CO2
D Decrease in H+
A

C Increase in CO2

57
Q
57 What happens when both carotid arteries are clamped?
A Only arterial BP increases
B Arterial BP decreases
C No change in arterial BP
D Heart Rate and arterial BP increases
A

D Heart Rate and arterial BP increases

58
Q
58 "A 28 year-old athlete is noted to have a baseline heart rate of 55 beats per minute, which his trainer attributes to excellent parasympathetic (vagal) tone. This parasympathetic effect, among other things, increases the acetylcholine (ACh) released in the sinoatrial (SA) and atrioventricular (AV) nodes. ACh released into the AV node causes which of the following in the AV fibers?"
A Prolonged PR interval
B Increased conduction velocity
C Increased cardiac contractility
D Widenig of the QRS comples
A

A Prolonged PR interval

59
Q
59 An experimental drug decreases cardiac rate by producing hyperpolarization in the pacemaker cells. To which receptors will this drug MOST probably bind to?
A Nicotinic receptors
B Alpha adrenergic receptors
C Muscarinic receptors
D Beta adrenergic receptors
A

C Muscarinic receptors

60
Q

60 Which of the following will BEST provide a sufficient amount of calcium to activate the contractile mechanism of the myocardial cell?
A The calcium-induced calcium release channel
B The amount of calcium in the sarcoplasmic reticulum
C The inward sodium current during the plateau phase of the myocardial action potential
D The number of open calcium channels

A

A The calcium-induced calcium release channel

61
Q
61 The plasma colloid osmotic pressure is derived from pressure from dissolved proteins and \_\_\_\_\_\_\_\_\_\_\_\_\_.
A "Anions (HCO3, Cl)"
B Lymph
C "Cations (Na, K)"
D "Gases (O2, CO2)"
A

C “Cations (Na, K)”

62
Q

62 What will be the resistance to blood flow if two vessels are connected in parallel?
A It will be more than the resistance of vessels connected in series
B It will be the sum of their individual resistances
C It will be less than the resistance of either vessel alone
D None of the choices is correct

A

C It will be less than the resistance of either vessel alone

63
Q

63 Which of the following correctly describes a second degree AV block?
A PR interval is shortened
B NOT all atrial impulses are conducted
C Rate of ventricular contraction is increased
D Dissociation between atrial and ventricular contractions

A

B NOT all atrial impulses are conducted

64
Q

64 A increased availability of acetylcholine will produce cardiac effects which result in which of the following?
A Decrease in the force of ventricular contraction and a slower conduction velocity
B Decrease in heart rate and a slower rate of conduction
C Increase in the force of atrial contraction and a faster rate of conduction
D Decrease in heart rate with a corresponding decrease of A-V or P-R interval

A

B Decrease in heart rate and a slower rate of conduction

65
Q
65 In which of the following situations do you expect maximal blood flow to the skeletal muscles?
A Jogging
B Lifting weights
C Climbing stairs
D Swimming
A

D Swimming

66
Q
66 What will be the Mean Arterial Pressure of an individual whose Arterial Blood Pressure is 140/80 mmHg?
A 120 mmHg
B 150 mmHg
C 90 mmHg
D 100 mmHg
A

D 100 mmHg

67
Q

67 Eisenmengerization occurs when:
A Pulmonary hypertension from shunt overload leads to right to left shunt
B A congenital ventricular septal defect closes at 1 year
C Atrial fibrillation occurs in Tetralogy of Fallot
D RV compliance exceeds LV compliance causing right to left shunt

A

A Pulmonary hypertension from shunt overload leads to right to left shunt

68
Q
68 A channel blocker like Verapamil decreases the influx of calcium in the cardiac myocytes during which phase of the ventricular action potential?
A Phase 1
B Phase 2
C Phase 3
D Phase 4
A

B Phase 2

69
Q

69 “STATEMENT A: The aortic & pulmonary valves snap to a closed position due to the high pressure in the arteries at the end of systole, in contrast to the softer closure of the AV valves. STATEMENT B: The aortic & pulmonary valves are subjected to much greater mechanical abrasion than are the A-V valves. “
A Both statements are true
B “The first statement is true, but the second statement is false”
C “The first statement is false, but the second statement is true”
D Both statements are false

A

A Both statements are true

70
Q
70 "In accordance with the Frank-Starling's principle, the greater the stretch of the cardiac muscle during ventricular filling, the greater the cardiac output. Which of the following will NOT significantly affect the efficacy of these mechanism?"
A Decreased Afterload
B Pre-load
C Myocardial Contractility
D Cardiac rate within physiologic limits
A

A Decreased Afterload

71
Q
71 Which of the following increases central venous pressure?
A Vasoactive intestinal peptide
B Kallidin
C Bradykinin
D Adenosine
A

A Vasoactive intestinal peptide

72
Q
72 Which of the following increases central venous pressure?
A Hemorrhage
B Heart Failure
C Tachycardia
D Dehydration
A

B Heart Failure

73
Q
73 Coronary blood flow is primarily regulated by \_\_\_\_\_\_\_\_\_\_\_\_\_\_.
A Sympathetic nervous system
B Epinephrine
C Metabolic demand
D Parasympathetic nervous system
A

C Metabolic demand

74
Q

74 The chronotropic property of the SA node action potential is due to which of the following?
A Increased potassium efflux at phase 3
B Increased calcium influx at phase O
C Increased potassium efflux and sodium leakage at phase 4
D Continuous calcium influx at phase 3

A

C Increased potassium efflux and sodium leakage at phase 4

75
Q

What is the reason for increased systemic resistance after the first breath at birth?
A Closure of ductus arteriosus
B Decreased pulmonary vascular resistance
C Closure of foramen ovale
D Loss of blood flow through placenta

A

D Loss of blood flow through placenta

76
Q

76 “During the reduced ejection phase of the left ventricle, which of the following is TRUE?”
A Aortic flow velocity reaches maximum
B Left atrial pressure is falling
C Left ventricular pressure is constant
D Aortic flow velocity is rapidly decrementing

A

D Aortic flow velocity is rapidly decrementing

77
Q
77 Which of the following mediators act in the long-term control of blood pressure?
A Carotid sinus
B Aldosterone
C Carotid bodies
D Adenosine
A

B Aldosterone

78
Q

78 “A cardiac patient is led to strain very hard, utilizing the Valsalva’s maneuver. Which of the following may take place?”
A There will be a corresponding reduction in venous inflow causing a marked fall in cardiac output
B The intra-thoracic pressure will increase and compress the heart
C The intra-thoracic pressure will decrease thereby diminishing venous return
D None of the choices is correct

A

A There will be a corresponding reduction in venous inflow causing a marked fall in cardiac output

79
Q

79 “Unlike skeletal muscles, why do cardiac muscles NOT exhibit tetanization?”
A Because the electrical refractory period and the mechanical contractile response are of almost equivalent duration
B Because cardiac muscle have long mechanical refractory period
C Because the mechanical contractile event is usually shorter than the duration of the electrical depolarization
D Because of the increased calcium release during contraction

A

A Because the electrical refractory period and the mechanical contractile response are of almost equivalent duration

80
Q
80  "A 20-year old comes for cardiac check-up. While monitoring her ECG, you ask he to lie down on the examination table and notice that there is a brief increase in her heart rate as soon as she lies down. What is most likely responsible for this transient heart rate change?"
A Baroreceptor reflex
B Bainbridge reflex
C Chemoreceptor reflex
D CNS ischemic response
A

B Bainbridge reflex

81
Q
81 When does maximal blood flow to the coronary vessels happen?
A Atrial relaxation
B Ventricular contraction
C Ventricular relaxation
D Atrial contraction
A

Ventricular relaxation

82
Q

82 Mr Tan’s blood pressure rose from 110/70 to 160/90 after seeing his daughter’s boyfriend. What will be the effect to his brain blood flow of this acute rise in BP?
A Decreases
B Increases
C Stays the same
D No correlation between BP and brain blood flow

A

C Stays the same

83
Q
83 "If the cardiac rate increases to 210 beats per minute, which of the following will increase?"
A End-diastolic volume
B End-systolic volume
C Stroke volume
D Ejection fraction
A

B End-systolic volume

84
Q

84 Which of the following is a characteristic of cutaneous circulation?
A Neural mechanisms are its most important regulators.
B Increased environmental temperatures constrict skin blood vessels
C AV anastomoses that help to control heat flow are found in the entire skin of the body.
D Activation of sweat glands help to conserve heat.

A

A Neural mechanisms are its most important regulators.

85
Q
85 At which cerebral PO2 does cerebral vasodilation occur?
A 40 mmHg
B 10 mmHg
C 20 mmHg
D 30 mmHg
A

D 30 mmHg

86
Q
86 Which of the following mechanisms best explains the character of blood flow distal to a site of arterial stenosis?
A Frank Starling's Law
B Law of La Place
C Reynolds number
D Bernoulli's Principle
A

C Reynolds number

87
Q

87 The SA nodal cells have an unstable resting potential of 70-mv. What will happen if the resting potential was shifted to 90-mv?
A The heart rate will decrease
B The rate of rise of the pacemaker potential will be slower
C It will take sooner to reach the threshold potential
D The heart rate will increase because of a faster rate of depolarization

A

A The heart rate will decrease

88
Q

88 “In the Poiseuille’s Equation, which of the following is TRUE?”
A Flow rate & length of the vessel are inversely proportional
B Flow rate & viscosity are directly proportional
C Flow rate & radius of the vessel are inversely proportional
D Flow rate is inversely proportional to pressure gradient

A

A Flow rate & length of the vessel are inversely proportional

89
Q
89 The liver is an organ of high blood flow and very low vascular resistance. What is the estimated pressure in the hepatic artery?
A 9 mmHg
B 6 mmHg
C 3 mmHg
D 0 mmHg
A

D 0 mmHg

90
Q

90 The cardiac muscle is characterized by which of the following?
A It shows the same graphic relation between initial fiber length and total tension as skeletal muscles
B It exhibits tetanic contraction
C It’s action potential shows a prepotential that is associated with the leakiness of the membrane to sodium ions
D It has a more developed sarcoplasmic reticulum than that of the skeletal muscles.

A

A It shows the same graphic relation between initial fiber length and total tension as skeletal muscles

91
Q

91 Which of the following is TRUE of the incisura or dicrotic notch of the aortic pressure pulse?
A It is magnified by aortic regurgitation
B It is indicative of cardio-vascular abnormality
C It is coincides with the second heart sound
D “It coincides with the ““v”” wave of the atrial pressure pulse”

A

C It is coincides with the second heart sound

92
Q

92 “In an EKG, which is the most variable waveform?”
A The deflection produced by atrial depolarization
B The deflection produced by ventricular repolarization
C The deflection produced by ventricular repolarization
D The ST segment

A

B The deflection produced by ventricular repolarization